Laplace transform with unit step function

Click For Summary
The discussion revolves around taking the Laplace transform of tH(t), where H(t) is the unit step function. The correct result for L{tH(t)} is indeed 1/s^2, contrary to the book's claim of 1/s. For the second problem involving e^{-t}H(t) - e^{-t}H(t-1), manipulation techniques are discussed, particularly using the identity e^{-t} = e^{-(t-1)}e^{-1}. Participants emphasize the importance of correctly factoring and manipulating exponential terms to achieve the desired forms for the Laplace transform. Overall, clarity in applying Laplace transform properties and manipulation of functions is essential for solving these problems effectively.
Juanriq
Messages
39
Reaction score
0

Homework Statement


I'm trying to take the laplace transfrom of t H(t) where H(t) is the unit step function. Also, in a separate problem I get e^{-t} H(t) - e^{-t}H(t-1) and I am wondering how to manipulate it properly

Homework Equations

L \{ f(t-a) H(t-a) \} = e^{as}F(s)



The Attempt at a Solution

For the first part, I thought that L \{ t H(t) \} should just give \frac{1}{s^2} back out, but the answer key in the book I'm using says that it is just \frac{1}{s}.



For the second part of my question, I know we have to manipulate the exponential, but how would I manipulate them? For instance, I want e^{t}H(t), but can't I only multiply by a constant? Obviously e^{-t}e^{2t}H(t) would do what I want, but now I'm introuducing something I can't factor outside the transform. Similarly for the second, e^{2t-1}e^{-t} [\latex] would do the trick... Any help is appreciated!
 
Physics news on Phys.org
Your book is wrong about L[tH(t)]. Your answer is right.

For the second problem, use the fact that e-t=e-(t-1)e-1.
 
Question: A clock's minute hand has length 4 and its hour hand has length 3. What is the distance between the tips at the moment when it is increasing most rapidly?(Putnam Exam Question) Answer: Making assumption that both the hands moves at constant angular velocities, the answer is ## \sqrt{7} .## But don't you think this assumption is somewhat doubtful and wrong?

Similar threads

Replies
2
Views
2K
  • · Replies 1 ·
Replies
1
Views
1K
  • · Replies 3 ·
Replies
3
Views
3K
  • · Replies 10 ·
Replies
10
Views
2K
  • · Replies 4 ·
Replies
4
Views
3K
Replies
2
Views
948
  • · Replies 3 ·
Replies
3
Views
1K
  • · Replies 1 ·
Replies
1
Views
928
Replies
1
Views
2K
  • · Replies 4 ·
Replies
4
Views
3K